Now you are in the subtree of Lecture Notes public knowledge tree. 

7.3. Правила Лопиталя

Теорема 7.6 (I правило о неопределенности $\frac{0}{0}$). Пусть функции $f$ и $g$ определены в окрестности точки $a$, $f(a) = g(a) = 0$, существуют $f'(a)$ и $g'(a) \neq 0$. Тогда существует

$$\lim \limits _{x \to a}\frac{f(x)}{g(x)} = \frac{f'(a)}{g'(a)}.$$

$\blacktriangle $ $\lim \limits _{x \to a} \frac{f(x)}{g(x)} = \lim \limits _{x \to a}\frac{f(x) - f(a)}{g(x) - g(a)} = \lim \limits _{x \to a}\frac{\frac{f(x) - f(a)}{x - a}}{\frac{g(x) - g(a)}{x - a}} = \frac{f'(a)}{g'(a)}$. $\blacksquare $

Доказанная теорема верна и для односторонних пределов и соответствующих односторонних производных.

Теорема 7.7 (II правило о неопределенности $\frac{0}{0}$). Пусть функции $f\colon (a, b) \to \mathbb {R}$,
$g\colon (a, b) \to \mathbb {R}$

  1. дифференцируемы на $(a, b)$,

  2. $\lim \limits _{\scriptscriptstyle x \to a + 0} f(x) = \lim \limits _{\scriptscriptstyle x \to a + 0} g(x) = 0$,

  3. $g'(x) \neq 0$ на $(a, b)$,

  4. $\exists \lim \limits _{\scriptscriptstyle x \to a + 0} \frac{f'(x)}{g'(x)} \in \overline{\mathbb {R}}$.

Тогда существует

$$\lim \limits _{\scriptscriptstyle x\to a + 0}\frac{f(x)}{g(x)} = \lim \limits _{\scriptscriptstyle x \to a + 0} \frac{f'(x)}{g'(x)}.$$

$\blacktriangle $ Доопределим $f$ и $g$ в точке $a$, положив $f(a) = g(a) = 0$. Тогда $\forall x \in (a, b)$ по Т7.4 Коши о среднем $\frac{f(x)}{g(x)} = \frac{f(x) - f(a)}{g(x) - g(a)} = \frac{f'(c)}{g'(c)}$, где $c \in (a, x), c = c(x)$.

Т.к. $\lim \limits _{\scriptscriptstyle x\to a+0} c(x) = a$ и $c(x) \neq a$, то по Т4.10 о замене переменной в пределе
$\lim \limits _{\scriptscriptstyle x\to a+0} \frac{f(x)}{g(x)} = \lim \limits _{\scriptscriptstyle c\to a+0} \frac{f'(c)}{g'(c)}$. $\blacksquare $

Замечание. Теорема также верна при $x\to a-0$ и $x\to a$.

Следствие. Пусть $f\colon (c, +\infty ) \to \mathbb {R},\ g\colon (c, + \infty ) \to \mathbb {R}\ (c > 0)$

  1. дифференцируемы на $(c, +\infty )$,

  2. $\lim \limits _{\scriptscriptstyle x \to +\infty } f(x) = \lim \limits _{\scriptscriptstyle x \to +\infty } g(x) = 0$,

  3. $g'(x) \neq 0$ на $(c, +\infty )$,

  4. $\exists \lim \limits _{\scriptscriptstyle x \to +\infty } \frac{f'(x)}{g'(x)} \in \overline{\mathbb {R}}$.

Тогда существует $\lim \limits _{\scriptscriptstyle x \to +\infty } \frac{f(x)}{g(x)} = \lim \limits _{\scriptscriptstyle x \to +\infty }\frac{f'(x)}{g'(x)}$.

$\blacktriangle $ Рассмотрим функции $\varphi \colon (0, \frac{1}{c}) \to \mathbb {R},\ \varphi (t) = f(\frac{1}{t})$ и $\psi \colon (0, \frac{1}{c}) \to \mathbb {R},\ \psi (t) = g(\frac{1}{t})$.

Функции $\varphi $ и $\psi $ дифференцируемы на $(0, \frac1c), \varphi '(t) = -f'(\frac1t) \frac1{t^2}$,    $\psi '(t) = -g'(\frac1t) \frac1{t^2} \neq 0$. По Т4.10 о замене переменной в пределе $\lim \limits _{\scriptscriptstyle t\to +0} \varphi (t) = \lim \limits _{\scriptscriptstyle x\to +\infty } f(x) = 0$,  $\lim \limits _{\scriptscriptstyle t\to +0} \psi (t) = \lim \limits _{\scriptscriptstyle x\to +\infty } g(x) = 0$ и

$\exists \lim \limits _{\scriptscriptstyle t\to +0} \frac{\varphi '(t)}{\psi '(t)} = \lim \limits _{\scriptscriptstyle x\to +\infty } \frac{f'(x)}{g'(x)}$. Тогда по Т7.7 (случай $a = 0$) $\exists \lim \limits _{\scriptscriptstyle t\to +0} \frac{\varphi (t)}{\psi (t)} = \lim \limits _{\scriptscriptstyle t\to +0} \frac{\varphi '(t)}{\psi '(t)}$. Откуда, учитывая что $\lim \limits _{\scriptscriptstyle t\to +0} \frac{\varphi (t)}{\psi (t)} = \lim \limits _{\scriptscriptstyle x\to +\infty } \frac{f(x)}{g(x)}$, получим $\lim \limits _{\scriptscriptstyle x\to +\infty } \frac{f(x)}{g(x)} = \lim \limits _{\scriptscriptstyle x\to +\infty } \frac{f'(x)}{g'(x)}$. $\blacksquare $

Замечание. Доказанное утверждение верно и при $x \to -\infty $.

Теорема 7.8 (правило о неопределенности $\frac\infty \infty $). Пусть функции $f\colon (a, b) \to \mathbb {R}$,

$g\colon (a, b) \to \mathbb {R}$

  1. дифференцируемы на $(a, b)$,

  2. $\lim \limits _{\scriptscriptstyle x \to a + 0} f(x) = \pm \infty ,\ \ \lim \limits _{\scriptscriptstyle x \to a + 0} g(x) = \pm \infty $,

  3. $g'(x) \neq 0$ на $(a, b)$,

  4. $\exists \lim \limits _{\scriptscriptstyle x \to a + 0} \frac{f'(x)}{g'(x)} \in \overline{\mathbb {R}}$.

Тогда существует

$$\lim \limits _{\scriptscriptstyle x \to a + 0} \frac{f(x)}{g(x)} = \lim \limits _{\scriptscriptstyle x \to a + 0} \frac{f'(x)}{g'(x)}.$$

$\blacktriangle $ Т.к. $\lim \limits _{x \to a + 0} g(x) = \pm \infty $, то можно дополнительно предположить, что $g(x) \neq 0$ на $(a, b)$. Пусть $x, x_0 \in (a, b), x < x_0$, тогда по Т7.4 Коши о среднем $\frac{f(x) - f(x_0)}{g(x) - g(x_0)} = \frac{f'(c)}{g'(c)}$ для некоторой точки $c \in (a, x_0)$.

Умножая это равенство на $\frac{g(x) - g(x_0)}{g(x)}$ и группируя члены, получаем, что

$$\frac{f(x)}{g(x)} = \frac{f'(c)}{g'(c)} + \frac{f(x_0)}{g(x)} - \frac{f'(c)}{g'(c)} \frac{g(x_0)}{g(x)}\ (*).$$

Пусть $\lim \limits _{x\to a+0} \frac{f'(x)}{g'(x)} = A \in \mathbb {R}$. Покажем, что $\lim \limits _{\scriptscriptstyle x\to a+0} \frac{f(x)}{g(x)} = A$.

Возьмём произвольное $\varepsilon >0$, найдём $\delta >0$, что $\forall t\in (a, a+\delta )\colon \left| \frac{f'(t)}{g'(t)} - A \right| < \frac\varepsilon 3$.

Выберем и зафиксируем $x_0 \in (a, a+\delta )$, тогда $\forall c\in (a, x_0), \left| \frac{f'(c)}{g'(c)} - A \right| < \frac\varepsilon 3$.

В силу условия $\lim \limits _{\scriptscriptstyle x\to a+0} \frac1{g(x)} = 0$. Поэтому $\exists \delta '\colon 0 < \delta ' \leqslant \delta $, что $\forall x\in (a, a+\delta ')$ выполняются неравенства $a < x < x_0$, $\left| \frac{f(x_0)}{g(x)} \right| < \frac\varepsilon 3$, $\left| \frac{g(x_0)}{g(x)} \right| < \frac\varepsilon {3|A| + \varepsilon }$.

Значит, при $x\in (a, a+\delta )\colon $ $\left| \frac{f(x)}{g(x)} - A \right| \stackrel{\scriptscriptstyle (*) - A}{=}$ $\left| \frac{f'(c)}{g'(c)} - A + \frac{f(x_0)}{g(x_0)} - \frac{f'(c)}{g'(c)} \frac{g(x_0)}{g(x)}\right| \leqslant $

$\leqslant \left|\frac{f'(c)}{g'(c)} - A\right| + \left|\frac{f(x_0)}{g(x_0)}\right| + \left|\frac{f'(c)}{g'(c)}\right| \left|\frac{g(x_0)}{g(x)}\right| < \frac\varepsilon 3 + \frac\varepsilon 3 + (|A| + \frac\varepsilon 3) \frac\varepsilon {3|A| + \varepsilon } < \varepsilon $,

т.е. $\lim \limits _{x\to a+0} \frac{f(x)}{g(x)} = A$.

Пусть $\lim \limits _{\scriptscriptstyle x\to a+0} \frac{f'(x)}{g'(x)} = \pm \infty $, тогда $\exists \lim \limits _{\scriptscriptstyle x\to a+0} \frac{g'(x)}{f'(x)} = 0$. Следовательно, по пункту I $\exists \lim \limits _{\scriptscriptstyle x\to a+0} \frac{g(x)}{f(x)}= \lim \limits _{\scriptscriptstyle x\to a+0} \frac{g'(x)}{f'(x)} = 0$. Значит $\lim \limits _{x\to a+0} \left| \frac{f(x)}{g(x)} \right| = +\infty $. По равенству $(*)$:

$$\frac{f(x)}{g(x)} = \left(1 - \frac{g(x_0)}{g(x)}\right) \frac{f'(c)}{g'(c)} + \frac{f(x_0)}{g(x)}.$$

Т.к. $\lim \limits _{\scriptscriptstyle x\to a+0} \frac{g(x_0)}{g(x)} = \lim \limits _{\scriptscriptstyle x\to a+0} \frac{f(x_0)}{g(x)}=0$, то существует интервал $(a, \alpha )$ на котором знак дроби $\frac{f(x)}{g(x)}$ совпадает со знаком $\frac{f'(c)}{g'(c)}$. Следовательно, $\lim \limits _{\scriptscriptstyle x\to a+0} \frac{f(x)}{g(x)} = \lim \limits _{\scriptscriptstyle x\to a+0} \frac{f'(x)}{g'(x)} = \pm \infty $. $\blacksquare $

Замечание. Данная теорема верна при $x\to a-0, x\to a$.

Следствие. См. предыдущее следствие, где 2) $\lim \limits _{\scriptscriptstyle x\to +\infty } f(x) =\pm \infty $, $\lim \limits _{\scriptscriptstyle x\to +\infty } g(x) =\pm \infty $.

$\blacktriangle $ Доказывается аналогично предыдущему следствию. $\blacksquare $

Замечание. Следствие верно и при $x\to -\infty $.

Примеры:($\frac\infty \infty $) Пусть $\alpha >0, a>1$, тогда $\lim \limits _{\scriptscriptstyle x\to +\infty } \frac{\ln x}{x^\alpha } =$ $\lim \limits _{\scriptscriptstyle x\to +\infty } \frac{\frac1x}{\alpha x^{\alpha - 1}} = \lim \limits _{\scriptscriptstyle x\to +\infty } \frac1{\alpha x^\alpha } = 0$.

$\lim \limits _{\scriptscriptstyle x\to +\infty } \frac{x^\alpha }{a^ x} =$ $\lim \limits _{\scriptscriptstyle x\to +\infty } \frac{ \alpha (\alpha -1)\ldots (\alpha -k+1)x^{\alpha -k} }{ a^ x \ln ^ k a } = $ $\lim \limits _{\scriptscriptstyle x\to +\infty } \frac{ \alpha (\alpha -1)\ldots (\alpha -k+1) }{x^{k-\alpha } a^ x \ln ^ k a } = 0$, где $k = [\alpha ] + 1$ (применили правило Лопиталя $k$ раз).

Замечание. Если предела $\lim \limits _{x\to a} \frac{f'(x)}{g'(x)}$ не существует, то это не означает, что не существует предела $\lim \limits _{x\to a} \frac{f(x)}{g(x)}$. Например: $\frac{f(x)}{g(x)} = \frac{x + \sin x}{x}$ при $x\to +\infty $.